Tỷ lệ phân phối độc lập nào cho phân phối bình thường?


12

Tỷ lệ của hai phân phối bình thường độc lập cho phân phối Cauchy. Phân phối t là phân phối chuẩn chia cho phân phối chi bình phương độc lập. Tỷ lệ của hai phân phối chi bình phương độc lập cho phân phối F.

Tôi đang tìm kiếm một tỷ lệ phân phối liên tục độc lập cung cấp cho một biến ngẫu nhiên phân phối chuẩn với trung bình μ và phương sai σ2 ?

Có lẽ có một bộ vô số các câu trả lời có thể. Bạn có thể cho tôi một số trong những câu trả lời có thể? Tôi đặc biệt đánh giá cao nếu hai phân phối độc lập có tỷ lệ được tính là giống nhau hoặc ít nhất có phương sai tương tự nhau.


2
Mặc dù bài viết Wikipedia về phân phối tỷ lệ không cung cấp các ví dụ về trường hợp mà bạn tìm kiếm, nhưng đây là một bài đọc thú vị.
Avraham

2
Một trường hợp khá đặc biệt là một tiêu chuẩn bình thường và Y độc lập ± 1 mỗi trường hợp có xác suất 1XY±1 , rồiX,YX12XY có cùng giá trị trung bình và phương sai vàXXY thường được phân phối. XY
Henry

1
" Tỷ lệ của hai phân phối chi bình phương độc lập cho phân phối F " --- tốt, không hoàn toàn. Nó cung cấp một bản phân phối beta-Prime. Để có được một F, bạn cần chia tỷ lệ cho mỗi hình vuông bằng df của nó.
Glen_b -Reinstate Monica

2
Một số điều khiến tôi hoàn toàn không tin rằng nó nhất thiết có thể đáp ứng tất cả các điều kiện của bạn.
Glen_b -Reinstate Monica

1
lấy việc tạo ra phương thức biến thông thường (ví dụ Box-Muller) làm ví dụ (sử dụng phương pháp vòng tròn) tôi sẽ nói rằng không có tỷ lệ phân phối thống nhất nào phân phối bình thường (giả sử phân phối thống nhất được yêu cầu)
Nikos M.

Câu trả lời:


5

Hãy nơiEcó phân phối mũ với trung bình2σ2Z=±1với xác suất bằng nhau. HãyY2=1/Y1=ZEE2σ2Z=±1 nơiB~Beta(0,5,0,5). Giả sử(Z,E,B)là độc lập lẫn nhau, sau đóY1không phụ thuộc vàoY2Y1/Y2~Bình thường(0,σ2). Do đó chúng ta cóY2=1/BBBeta(0.5,0.5)(Z,E,B)Y1Y2Y1/Y2Normal(0,σ2)

  1. độc lập với Y 2 ;Y1Y2
  2. Cả hai liên tục; như vậy mà
  3. Y1/Y2Normal(0,σ2) .

Tôi chưa tìm ra cách để có được một . Đó là khó khăn hơn để xem làm thế nào để làm điều này kể từ khi vấn đề giảm để việc tìm kiếm MộtB là độc lập như vậy Một - B μNormal(μ,σ2)AB khó hơn một chút so với tạoA/BBình thường(0,1)choABđộc lập.

ABμBNormal(0,1)
A/BNormal(0,1)AB

1
Nếu điều này là đúng, điều này thật tuyệt vời.
Neil G

2
@NeilG đó là sự thật; sản phẩm của phiên bản beta và cấp số nhân của tôi là một gamma có hình dạng 1/2 (vì cách bạn có thể xây dựng bản beta và một gamma độc lập bằng cách sử dụng gamma). Sau đó, căn bậc hai của nó là một nửa bình thường sử dụng thực tế là bình phương của một bình thường là chi bình phương.
anh chàng

1
Gần đây chúng tôi đã có một câu hỏi yêu cầu một sản phẩm gồm hai biến được phân phối bình thường (tôi không thể tìm lại được). Câu hỏi đó có một nhận xét hoặc câu trả lời liên quan đến biến đổi Box-Muller , tính toán phân phối bình thường (hay chính xác hơn là phân phối chuẩn bivariate) từ sản phẩm của hai biến phân phối thống nhất được chuyển đổi. Câu trả lời này liên quan rất nhiều đến điều đó nhưng lại nghịch đảo một trong những biến đó trong phép biến đổi Box-Muller. cc: @kjetilbhalvorsen
Sextus Empiricus

1

Tôi đặc biệt đánh giá cao nếu hai phân phối độc lập có tỷ lệ được tính là như nhau 

không khả năng rằng một biến bình thường có thể được viết dưới dạng tỉ số của hai biến độc lập với cùng phân phối hoặc gia đình phân phối (ví dụ như F-phân phối đó là tỷ số của hai quy mô χ2 biến phân phối hoặc Cauchy-phân phối đó là tỷ lệ của hai biến phân phối bình thường với giá trị trung bình bằng 0).

  • Giả sử rằng: đối với bất kỳ A,BF nơi F là phân phối hoặc phân phối cùng gia đình chúng tôi có

    X=ABN(μ,σ2)

  • Chúng ta cũng phải có khả năng đảo ngược AB (nếu một biến bình thường có thể được viết dưới dạng tỷ lệ của hai biến độc lập có cùng phân phối hoặc họ phân phối thì thứ tự có thể được đảo ngược)

    1X=BAN(μ,σ2)

  • Nhưng nếu XN(μ,σ2) sau đó X1N(μ,σ2) không thể là sự thật (nghịch đảo của một biến phân phối bình thường là không một biến phân phối bình thường).

Kết luận rộng hơn: Nếu các biến trong bất kỳ họ phân phối FX có thể được viết dưới dạng tỷ lệ của các biến trong một phân phối khác FY thì đó phải là họ FX được đóng theo lấy đối ứng (nghĩa là đối với bất kỳ biến nào có phân phối FX phân phối đối ứng của nó cũng sẽ nằm trong FX ).

Ví dụ: nghịch đảo của một biến phân phối Cauchy cũng được phân phối Cauchy. Nghịch đảo của biến phân phối F cũng được phân phối F.

  • Điều này 'nếu' không phải là 'iff', điều ngược lại là không đúng. Khi X1/X ở cùng một họ phân phối thì không phải lúc nào cũng có thể được viết dưới dạng phân phối tỷ lệ với người đề cử và mẫu số từ cùng một họ phân phối.

    Ví dụ: Chúng ta có thể tưởng tượng các gia đình phân phối mà với bất kỳ X nào trong gia đình chúng ta có 1/X trong cùng một gia đình nhưng chúng ta không có P(X=1)=0 . Đây là mâu thuẫn với thực tế là cho một phân phối tỷ lệ mà mẫu số và đề cử có cùng một phân phối chúng ta phải có P(X=1)0 (và một cái gì đó tương tự có thể được biểu diễn cho các bản phân phối liên tục như không thể thiếu dọc theo dòng X / Y = 1 trong một biểu đồ phân tán của X, Y có mật độ khác không khi X và Y có cùng phân phối và độc lập).


Đừng nhìn thấy nó. Dường như với tôi rằng chỉ vì B / C bình thường mà không tạo ra A /A/DB/CA/DB/C

Tốt hơn. Bây giờ nó có ý nghĩa.
Carl

1
I don't understand how the second statement follows from the first. If there exists some A,B such that their quotient is normal, why does it follow that their quotient in the other order should also be normal? The question didn't ask for a distribution family such that the quotient of all pairs of elements is normal.
Neil G

1
I don't understand what you're saying. Ideally, your answer would be a coherent argument without requiring someone to read the edits. Right now, it seems like your second statement ("we must also have") doesn't follow from the first.
Neil G

1
@kjetilbhalvorsen how does it need to be revised? I have answered the part of the question that specifies "I would particularly appreciate if the two independent distributions which ratio is computed are the same". I do not see how the answer by guy relates to it.
Sextus Empiricus

0

Well, here is one but I will not prove it, only show it in simulation.

Make two beta distributions with equal large shape parameters Beta(200,200) (here, n=40,000), subtract 1/2 from the x-values of one of them and call it "numerator." That gives us a PDF that has a maximum range of (12,12), but because the shape parameters are so large, we never get to the maximum values of the range. Here is a histogram of an n=40,000 "numerator" enter image description here

Next, we call the second beta distribution "denominator" without subtracting anything, so it has the usual beta distribution range of (0,1) and one of those looks like this

enter image description here

Again, because the shapes are so large, we do not approach the maximum range with the values. Next we plot the quotient numeratordenominator as a PDF with the superimposed normal distribution.

enter image description here

Now in this case the normal distribution result has μ0.0000204825,σ0.0501789 and tests for normality that look like this

(StatisticP-ValueAnderson-Darling0.7997860.481181Baringhaus-Henze1.405850.0852017Cramér-von Mises0.1231450.482844Jarque-Bera ALM4.481030.106404Kolmogorov-Smirnov0.004523280.386335Kuiper0.007980630.109127Mardia Combined4.481030.106404Mardia Kurtosis1.538490.123929Mardia Skewness2.093990.147879Pearson χ2134.3530.571925Watson U20.1138310.211187)

In other words, we cannot prove the ratio is not normal even trying very hard to do so.

Now why? Intuition on my part, which I have in overabundance. Proof left to reader, if any exists (maybe via limit of method of moments, but again that is just intuition).

Hint: If I use only Beta(20,20) in denominator and Beta(20,20)12 in the numerator and I get Student's t with μ0.000251208,σ0.157665,df33.0402

enter image description here

StatisticP-ValueAnderson-Darling0.2752620.955502Cramér-von Mises0.03511080.956524Kolmogorov-Smirnov0.003209360.804486Kuiper0.005565010.657146Pearson χ2145.0770.323168Watson U20.03510420.878202

Another hint N(0,1)N(10,1/1000) Student's t μ0.0000535722,σ0.0992765,df244.154

enter image description here

(StatisticP-ValueAnderson-Darling0.5016770.745102Cramér-von Mises0.06968240.753515Kolmogorov-Smirnov0.003556880.692225Kuiper0.006083820.501133Pearson χ2142.880.370552Watson U20.06032070.590369)

5
You are clearly very close to a normal distribution. However, that isn't at all the same thing as having a normal distribution, and I don't believe the ratio of a centered symmetric beta to an ordinary symmetric beta with the same parameters is ever to be actually normal. I'd be very interested in being wrong about this though.
Glen_b -Reinstate Monica

2
Your solution definitely is not Normal. You could generalize this approach: take any distribution that is approximately Normal and divide it by a distribution with its probability concentrated near a nonzero number. The result (obviously) will be close to Normal--but it still will not be Normal. Applying a bunch of tests is unconvincing because all it shows it that you didn't generate sufficiently large samples to demonstrate the non-Normality.
whuber

1
@whuber At 108 one could also show in normal machine precision that noise will cause anything to be not normal. I do not have a super computer to do that in extended precision. What you could show, and why I wanted you to look at this, was to prove or disprove these things mathematically, not just criticize around the edges with unachievable goals.
Carl

2
Let me get to the heart of the matter, then: (1) disproving normality is a simple exercise in integral approximation--no need to give the details here. You can, e.g., readily prove the 200th moment is infinite. (2) Your answer confuses distributions with samples. It is this fundamental confusion that I object to; it's the reason why I think this answer is more misleading than helpful. BTW, I did not write my last comment lightly: I performed that test. I did it not with a supercomputer, but with a decade-old PC workstation, and the whole process took just seconds.
whuber

1
@whuber Which approximation are you testing? The first, the second or the third? BTW, if they are only approximations, so be it. I suggest only that in the limiting case that they might be exact. All of statistics is an approximation so I do not share your apprehension.
Carl

-3

I imagine there are many possibilities. Here there is one that I can think of. It is known (Zolotarev) that, given X1G,X2G two standard normal distributed r.v., and XγC a Cauchy distributed r.v.

X1GX2G=XγC

Then, by Duality of the Stable distribution, we know that XγC1/X1/γC (where γ is the scale parameter of the Cauchy). So you get that the Normal distribution can be a result from a ratio between a Normal and a Cauchy:

X1G=X2G/X1/γC

for the desired μ I would just move both distributions to be centred there. (at μ). For the σ, in the mentioned wikipedia page about ratio distributions, there are the general formulas for the ratio of two normal distributions, you would just need to replace the scale factor of the Cauchy by its inverse value (γ1/γ).


4
Please test your hypothesis, either by explicit calculation of the ratio or via simulation. Either will show that your claim is incorrect. The error lies in assuming that distribution ratios can be "canceled" to "solve for" the numerator.
whuber

1
hm, it is true that the passage of the X2G to the right is shady. I'll check.
chuse
Khi sử dụng trang web của chúng tôi, bạn xác nhận rằng bạn đã đọc và hiểu Chính sách cookieChính sách bảo mật của chúng tôi.
Licensed under cc by-sa 3.0 with attribution required.